Đến nội dung

kerry0111 nội dung

Có 20 mục bởi kerry0111 (Tìm giới hạn từ 27-04-2020)


Sắp theo                Sắp xếp  

#403611 ĐỀ THI HỌC SINH GIỎI LỚP 10 CHUYÊN SƯ PHẠM (ngày 1)

Đã gửi bởi kerry0111 on 10-03-2013 - 12:57 trong Thi HSG cấp Tỉnh, Thành phố. Olympic 30-4. Đề thi và kiểm tra đội tuyển các cấp.

Bài 3:
Cho tam giác $ABC$ không cân,nội tiếp đường tròn $(O)$ và ngoại tiếp đường tròn $(I)$. $(I)$ thứ tự tiếp xúc $BC,CA,AB$ tại $A_0,B_0,C_0$. Đường tròn $(O_a)$ thuộc nửa mặt phẳng bờ $BC$ chứa $A$,tiếp xúc với $BC$ tại $A_0$ và tiếp xúc trong với $(O)$ tại $A_1$. $A_2$ là giao điểm của $AA_1$ và $BC$. Tương tự có $B_1;C_1$. Chứng minh rằng:
a,$\widehat{AA_1I}=\widehat{BB_1I}=\widehat{CC_1I}=90^{\circ}$
b,$A_2;B_2;C_2$ cùng thuộc một đường thẳng và đường thẳng đó vuông góc với $OI$.


l
a, dễ thấy $J$ là điểm chính giữa cung $BC$ k chứa $A$ thì $J, A_0, A_1$ thẳng hàng

và $\widehat{A_1AJ}=\widehat{A_1A_0B}$ nên ta chỉ cần cm $\widehat{AIA_1}=\widehat{IA_0E}$

hay $\widehat{IA_0J}=\widehat{JIA_1}$

mà $JI^2=JB^2=JA_0.JA_1$

ta có đpcm

b, $\widehat{A_2AI}=\widehat{A_1A_0A_2}\Rightarrow A_2A_1.A_2A=A_2I^2\Rightarrow A_2O^2-A_2I^2=R^2$

xd 2 hệ thức tt và theo định lý 4 điểm ta có đpcm



#403604 ĐỀ THI HSG LỚP 10 CHUYÊN SƯ PHẠM (ngày 2)

Đã gửi bởi kerry0111 on 10-03-2013 - 12:38 trong Thi HSG cấp Tỉnh, Thành phố. Olympic 30-4. Đề thi và kiểm tra đội tuyển các cấp.

b, $EK=2EO_1.sinEXK=2AE.sin\frac{BAD}{2}.sinAEF$

$Fl=2FO_2.sinFYL=2AF.sin\frac{CAD}{2}.sinAFE$

$\Rightarrow$ đpcm



#403601 \[ \phi(\sigma(2^x))=2^x \]

Đã gửi bởi kerry0111 on 10-03-2013 - 12:30 trong Số học

Giải phương trình nghiệm nguyên:
\[ \phi(\sigma(2^x))=2^x \]

Albania BMO 2009


$\phi \left ( \sigma \left ( 2^x \right ) \right )=2^x$

$\Leftrightarrow \phi \left ( 2^n-1 \right )=2^{n-1}$ với $n=x+1$

$\Rightarrow 2^n-1|2^{2^{n-1}}-1$

$\Rightarrow n|2^{n-1}$

$\Rightarrow n=2^m$

$\Rightarrow \phi (2^{2^m}-1)=2^{2^m-1}$

$\Rightarrow 2^{2^m-1}=\phi \left ( F_{m}-2 \right )=\prod_{k=0}^{m-1}\phi \left ( F_{k} \right )\leq \prod_{k=0}^{m-1}2^{2^{k}}=2^{2^m-1}$

$\Rightarrow m-1 \in \left \{ 0;1;2;3;4 \right \}$



#382375 Bài toán điểm Naghen và công thức Euler.

Đã gửi bởi kerry0111 on 31-12-2012 - 21:28 trong Hình học

theo công thức euler ta cần cm $R^2-d^2=4Rr-4r^2$

ta cũng có hệ thức $R^2-ON^2=xyc^2+yza^2+zxb^2$

$(x=\frac{S_{NBC}}{S_{ABC}}=\frac{p-a}{p}, y=\frac{S_{NCA}}{S_{ABC}}=\frac{p-b}{p}, z=\frac{S_{NAB}}{S_{ABC}}=\frac{p-c}{p})$

$xyc^2+yza^2+zxb^2=\frac{\sum a^2(a+b-c)(a+c-b)}{\left ( \sum a \right )^2}=\frac{\sum a^4-2\sum a^2b^2+2\sum a^2bc}{\left ( \sum a \right )^2}$

$4Rr-4r^2=\frac{abc}{p}-\frac{4S^2}{p^2}=\frac{2abc-\prod (a+b-c)}{\sum a}=\frac{\sum a^4-2\sum a^2b^2+2\sum a^2bc}{\left ( \sum a \right )^2}$

tóm lại ta có đpcm



#382296 $$\frac{1}{8a^2+bc}+\frac{1...

Đã gửi bởi kerry0111 on 31-12-2012 - 17:05 trong Bất đẳng thức - Cực trị

Bài toán 2.
Chứng minh rằng $\forall a,b,c>0$ ta luôn có 2 chiều bất đẳng thức:
$$\frac{3(a^2+b^2+c^2)}{a+b+c}\geq \sqrt{a^2-ab+b^2}+\sqrt{b^2-bc+c^2}+\sqrt{c^2-ca+a^2}\geq \sqrt{3(a^2+b^2+c^2)}$$


CM : $\sqrt{a^2-ab+b^2}+\sqrt{b^2-bc+c^2}+\sqrt{c^2-ca+a^2}\geq \sqrt{3(a^2+b^2+c^2)}$

$\Leftrightarrow 2\sum a^2-\sum ab+2\sum \sqrt{(a^2-ab+b^2)(b^2-bc+c^2)}\geq 3\sum a^2$

$\Leftrightarrow 2\sum \sqrt{(a^2-ab+b^2)(b^2-bc+c^2)}\geq \sum a^2+\sum ab$

$2\sum \sqrt{(a^2-ab+b^2)(b^2-bc+c^2)}$


$\geq \sum \sqrt{(a^2+b^2)(b^2+c^2)}$

$\geq \sum a^2+\sum ab$


đpcm

còn vế thứ nhất sai thì phải :-"



#382022 $\sum \frac{1}{2a^{2}+bc}\g...

Đã gửi bởi kerry0111 on 30-12-2012 - 17:20 trong Bất đẳng thức - Cực trị

Cho các số thực không âm thỏa mãn $ab+bc+ca>0$
Chứng minh rằng
$\sum \frac{1}{2a^{2}+bc}\geq \frac{8}{(a+b+c)^{2}}$


$\sum \frac{1}{2a^2+bc}=\sum \frac{(b+c)^2}{(b+c)^2(2a^2+bc)}\geq \frac{4\left ( \sum a \right )^2}{\sum(b+c)^2(2a^2+bc) }$

do đó ta cần c/m

$\left ( \sum a \right )^4\geq 2\sum(b+c)^2(2a^2+bc)$

$\Leftrightarrow \sum a^4+2\sum_{sym} a^3b+4\sum a^2bc\geq 6\sum a^2b^2$

$($ đúng theo $Schur$ và $AM-GM$ $)$



#381969 $\frac{1}{4a^2-ab+4b^2}+\frac{1}...

Đã gửi bởi kerry0111 on 30-12-2012 - 13:59 trong Bất đẳng thức - Cực trị

Chứng minh với mọi a,b,c dương :nav:
$\frac{1}{4a^2-ab+4b^2}+\frac{1}{4b^2-bc+4c^2}+\frac{1}{4c^2-ca+4a^2}\geq \frac{9}{7(a^2+b^2+c^2)}$



chuẩn hóa $p=3$

bđt cần cm tđ vs

$\left ( 4\sum a^2-\sum ab \right )\left ( \sum \frac{1}{4a^2-ab+4b^2} \right ) \geq \frac{9\left ( 4\sum a^2-\sum ab \right )}{7\left ( \sum a^2 \right )}$

$\Leftrightarrow \sum \frac{4c^2-cb-ca}{4a^2-ab+4b^2}+\frac{9\left ( \sum ab \right )}{7\left ( \sum a^2 \right )}\geq \frac{15}{7}$

mà $\sum \frac{4c^2-cb-ca}{4a^2-ab+4b^2}\geq \frac{\left ( 4\sum a^2-2\sum ab \right )^2}{\sum (4c^2-cb-ca)(4a^2-ab+4b^2)}=\frac{(36-10q)^2}{40q^2-36q-70r}$

do đó ta cần c/m

$\frac{(36-10q)^2}{40q^2-36q-70r}+\frac{9q}{7(9-2q)}\geq \frac{15}{7}$

$\Leftrightarrow 2(40q^3+2394q^2-14661q+20412)+315(45-13q)r\geq 0$

nếu $q\leq \frac{9}{4}\Rightarrow 40q^3+2394q^2-14661q+20412=(4q-9)(10q^2+621q-2268)\geq 0$

$\Rightarrow 2(40q^3+2394q^2-14661q+20412)+315(45-13q)r\geq 0$

nếu $3\geq q\geq \frac{9}{4}$

do $r \geq \frac{4q-9}{3}$ nên $2(40q^3+2394q^2-14661q+20412)+315(45-13q)r\geq (q-3)(4q-9)(20q-63)\geq 0$

tóm lại bđt đc cm



#381505 Chứng minh rằng: $\frac{1}{r}= \frac{...

Đã gửi bởi kerry0111 on 29-12-2012 - 10:47 trong Hình học

Bài 2: Cho tam giác ABC không cân tại A, M là trung điểm của BC. Điểm I thoả mãn điều kiện IB = IC. Đường tròn (I, IA) theo thứ tự lại cắt AB, AC tại P, Q. PQ cắt BC tại K. AM lại cắt (I, IA) tại L. Chứng minh rằng KL tiếp xúc với (I, IA).


Hình đã gửi

$P_{B/(I)}=P_{C/(I)}\Rightarrow BE.BA=CQ.CA$

mà $\frac{KP}{KQ}.\frac{CQ}{CA}.\frac{BA}{BP}=1$

nên $\frac{KP}{KQ}=\left ( \frac{AC}{AB} \right )^2$

mặt khác $\frac{PL}{LQ}=\frac{sinMAB}{sinMAC}=\frac{AM.\frac{sinAMB}{AB}}{AM.\frac{sinAMC}{AC}}=\frac{AC}{AB}$

do đó $\frac{KP}{KQ}=\left ( \frac{PL}{LQ} \right )^2$

từ K kẻ tiếp tuyến $KL'$ của $(I)$ ( $L'$ thuộc cung $PQ$ chứa L )

thì $\frac{KP}{KQ}=\left ( \frac{PL'}{L'Q} \right )^2$

do đó $L \equiv L'$ hay ta có đpcm



#381172 Chứng minh rằng $O$ đối xứng với $O'$ qua $I$

Đã gửi bởi kerry0111 on 28-12-2012 - 12:17 trong Hình học

Bài toán: [Gin Mellkior] Cho tứ giác $ABCD$ nội tiếp đường tròn tâm $O$. Gọi $M$, $N$, $P$, $Q$ lần lượt là trung điểm của $AB$, $BC$, $CA$, $DA$. Từ $M$, $N$, $P$, $Q$ lần lượt kẻ các đường vuông góc xuống các cạnh đối diện.
a) Chứng minh rằng các đường vuông góc đó đồng quy tại $O'$.
b) Gọi $I$ là giao điểm của $MP$ và $NQ$. Chứng minh rằng $O$ đối xứng với $O'$ qua $I$.


gọi $ O' $ là giao của đt đi qua M vuông góc CD và đt đi qua P vuông góc AB

$\Rightarrow \diamond MO'PO$ là hình bình hành

$\Rightarrow \diamond QO'NO$ là hình bình hành

gọi $ O" $ là giao của đt đi qua Q vuông góc AD và đt đi qua N vuông góc BC

$\Rightarrow \diamond QO"NO$ là hình bình hành

do đó $O'\equiv O"$ hay ta có đpcm



#378651 $a\sqrt{b^2+4c^2}+b\sqrt{4a^2+c^2}+c\...

Đã gửi bởi kerry0111 on 18-12-2012 - 20:07 trong Bất đẳng thức - Cực trị




Bài toán 2:Cho các số thực không âm$a,b,c$. Chứng minh rằng:



$a\sqrt{b^2+4c^2}+b\sqrt{4a^2+c^2}+c\sqrt{a^2+4b^2}\leq$$\frac{3}{4}(a+b+c)^2$



k mất tính tq gs $c=min\left \{ a,b,c \right \}$

$\left ( \sum _{cyc}a\sqrt{b^2+4c^2} \right )^2\leq \left ( \sum _{cyc}a(3a+b+5c) \right )\left ( \sum _{cyc}\frac{a(b^2+4c^2)}{3a+b+5c} \right )$

do đó ta chỉ cần c/m $\sum _{cyc}\frac{a(b^2+4c^2)}{3a+b+5c} \leq \frac{3}{16}\left ( \sum a\right )^2$

$\Leftrightarrow 45\sum a^5+165\sum _{cyc}a^4b+69\sum _{cyc}ab^4+536\sum a^3bc-306\sum _{cyc}a^3b^2-18\sum _{cyc}a^2b^3-410\sum a^2b^2c\geq 0$

$\Leftrightarrow 3(a+5b)(15a^2+10ab+3b^2)(a-b)^2+Ac\geq 0$

trong đó $A=69a^4+(536b-18c)a^3-(410b^2+410bc+306c^2)a^2+(536b^3-410b^2c+436bc^2+165c^3)a+165b^4-306b^3c-18b^2c^2+69bc^3+45c^4$

sd $c=min\left \{ a,b,c \right \}$ dễ dàng c/m đc $A\geq 0$




#378648 $M$ là trung điểm của $PQ$

Đã gửi bởi kerry0111 on 18-12-2012 - 19:52 trong Hình học

Cho đường tròn nội tiếp $(O)$ của tam giác $ABC$.Gọi $M$ là trung điểm $BC, AM$ cắt $(O)$ tại hai điểm $K$ và $L$($K$ nằm giữa $A$ và $L$).Qua $K$ kẻ đường thẳng song song với $BC$ cắt $(O)$ tại điểm thứ hai là $X$, Qua $L$ kẻ đường thẳng song song với $BC$ cắt $(O)$ tại điểm thứ hai là $Y$, $AX$ và $AY$ cắt $BC$ lần lượt tại $Q$ và $P$. Chứng minh rằng $M$ là trung điểm của $PQ$.


Hình đã gửi

gọi D, E, F là tiếp điểm của (O) với BC, CA, AB, I là điểm đồng quy của XY, KL, OD

$\frac{MP}{MQ}=\frac{\frac{YL.AM}{AL}}{\frac{KX.AM}{AK}}=\frac{IL}{IK}.\frac{AK}{AL}$

do đó ta cần c/m $(A, I, K, L)=-1$

mặt khác ta cũng có kết quả quen thuộc là I thuộc EF( có thể c/m =pp vectơ hoặc dùng hàng điểm điều hòa) và khi đó $(A, I, K, L)=-1$

tóm lại bt đc c/m



#378602 Chứng minh $K$ thuộc đường tròn ngoại tiếp tam giác $ABC$

Đã gửi bởi kerry0111 on 18-12-2012 - 17:31 trong Hình học

Hình đã gửi



gọi K là giao của $CP$ với $(O)$

$\widehat{PDB}=\widehat{CAB}=\widehat{PKB}$ $\Rightarrow G, P, D, K, B$ đồng viên

tt $F, P, E, K, A$ đồng viên

$\widehat{GPB}=\widehat{CGP}-\widehat{GBP}=\widehat{CAB}-\widehat{PAB}=\widehat{FAP}$

$\widehat{PKF}=\widehat{PAF}=\widehat{GPB}=\widehat{PGD}=\widehat{PKD}$

suy ra $K, D, F$ thẳng hàng

tt $K, E, G$ thẳng hàng

ta có đpcm



#378118 .CHứng minh $\frac{{SU.SP}}{{TU.TP...

Đã gửi bởi kerry0111 on 16-12-2012 - 19:52 trong Hình học

Cho tam giác ABC vuông tại A (AC>AB),có O là tâm đường tròn ngoại tiếp.Gọi la;lb là tiếp tuyến của (O) tại A,B.$S = la \cap BC;D = AC \cap lb;E = AB \cap DS;T = CE \cap la$.Hạ $EP \bot la,(P \in la)$.Gọi $Q = CP \cap (O);R = QT \cap (O);U = BR \cap la$.CHứng minh $\frac{{SU.SP}}{{TU.TP}} = \frac{{S{A^2}}}{{T{A^2}}}$


a/d Menelaus cho $\Delta STI$ với 2 cát tuyến $URB$ và $PCQ$

$\frac{US}{UT}.\frac{RT}{RI}.\frac{BI}{BS}=1$

$\frac{PS}{PT}.\frac{QT}{QI}.\frac{CI}{CS}=1$

từ đó dễ dàng -> đpcm

( ở trên dùng độ dài đại số)



#377557 $x^2+1\vdots y;y^3+1\vdots x^2$

Đã gửi bởi kerry0111 on 14-12-2012 - 18:45 trong Số học

Bài toán 2:
Tìm các số nguyên dương $x,y$ sao cho $x^2+1\vdots y;y^3+1\vdots x^2$


$y=1 \Rightarrow x=1$

$y=2 \Rightarrow x\in \left \{ 1;3 \right \}$

$y=3$ k có x t/m

$y>3$

đặt $x^2+1=y.k, y^3+1=q.x^2$

$\Rightarrow x^2|y^3k^3+k^3 \Rightarrow x^2|(x^2+1)^3+k^3\Rightarrow x^2|k^3+1$

gs $k> y$

$y^3+1=q(yk-1)\Rightarrow y^3-qky+q+1=0$

$y|q+1\Rightarrow q+1\geq y\Rightarrow q\geq y$

(dễ thấy $q+1\neq y$)

do đó $0\leq yq(y-k)+q+1< 1-q$ (vô lí)



#376182 $\sum \frac{1}{\frac{8}{5...

Đã gửi bởi kerry0111 on 09-12-2012 - 00:41 trong Bất đẳng thức - Cực trị

Cho $a,b,c\geq 0$ và $ab+bc+ac=1$
Chứng minh rằng

$\frac{1}{\frac{8}{5}.a^2+bc}+\frac{1}{\frac{8}{5}.b^2+ac}+\frac{1}{\frac{8}{5}.c^2+ab}\geq \frac{9}{4}$


nếu 1 trong 3 số a, b, c bằng 0

gs a=0 thì bđt hiển nhiên đúng theo AM-GM

$"=" \Leftrightarrow b=c=1$

nếu abc>0

đặt $ab=x, bc=y, ca=z$

bđt cần c/m tt $\sum \frac{x}{8x^2+yz}\geq \frac{9}{20\sum x}$

$\Leftrightarrow 800\prod_{cyc}(x-y)^2+1400\sum x^3y^3+xyz(980\sum_{sym}x^2y+567xyz)\geq 0$

(đúng)



#376175 $\dfrac{AN}{NC}=\dfrac{AI^2}...

Đã gửi bởi kerry0111 on 08-12-2012 - 23:38 trong Hình học

giúp mình với
cho (O;R) vẽ hai dây bất kì cắt nhau tại I là AB và CD, I nằm trong (O;R). Gọi M là trung điểm của AD. MI kéo dài cắt AC tại n.
chứng minh rằng
$\dfrac{AN}{NC}=\dfrac{AI^2}{IC^2}$


$\frac{AN}{NC}=\frac{S_{\Delta AIN}}{S_{\Delta NIC}}=\frac{S_{\Delta AIN}}{S_{\Delta MIB}}.\frac{S_{\Delta MID}}{S_{\Delta NIC}}=\frac{IA.IN}{IM.IB}.\frac{IM.ID}{IN.IC}=\frac{IA^2}{IC^2}$



#375911 $$\frac{x}{y+z}+\frac{y}...

Đã gửi bởi kerry0111 on 07-12-2012 - 22:42 trong Bất đẳng thức - Cực trị

Bài toán 1.
Ch0 3 số thực không âm $x,y,z$. Chứng minh rằng:
$$\frac{x}{y+z}+\frac{y}{x+z}+\frac{z}{x+y}+\frac{4(x+y)(y+z)(x+z)}{x^3+y^3+z^3}\geq 5$$


bđt cần cm tđ với

$\sum \frac{x}{y+z}-1+\frac{4\prod (x+y)}{\sum x^3}\geq 4$

$\Leftrightarrow \frac{\sum x^3+xyz}{\prod (x+y)}+\frac{4\prod (x+y)}{\sum x^3}\geq 4$

đúng theo AM-GM và $xyz\geq 0$

Bài toán 2.
Chứng minh rằng $\forall a,b,c\geq 0$ ta luôn có bất đẳng thức :
$$\frac{a}{b+c}+\frac{b}{a+c}+\frac{c}{a+b}+\frac{9(ab+bc+ca)}{a^2+b^2+c^2)}\geq 5$$


bđt cần cm tđ với

$\frac{\sum a^3+\sum a^2b+3abc}{\prod (a+b)}+\frac{9\sum ab}{\sum a^2}\geq 6$

a/d tt AM-GM thì ta chỉ cần cm

$(\sum a^3+\sum a^2b+3abc)(\sum ab)\geq (\sum a^2)\prod (a+b)$

$\Leftrightarrow abc(a^2+3ab+3ac+b^2+bc+c^2)\geq 0$

(đúng)



#373306 $(a^3b+b^3c+c^3a)(ab+ac+bc)\leq 16$

Đã gửi bởi kerry0111 on 28-11-2012 - 17:47 trong Bất đẳng thức - Cực trị


Bài toán 2: Cho $a,b,c\geq 0$ và $a+b+c=3$. Chứng minh rằng:

$(a^3b+b^3c+c^3a)(ab+ac+bc)\leq 16$



gs a là số nằm giữa b và c

$b(a+b)(b-a)(a-c)\geq 0\Rightarrow a^3b+b^3c+c^3a\leq a^2bc+ab^3+ac^3$

gs $c\leq a\leq b$

$a^2bc+ab^3+ac^3\leq a^2bc+ab^3+b^2c^2$

do đó $(a^3b+b^3c+c^3a)(ab+bc+ca)\leq (ab^2+bc^2+ca^2)(abc+ab^2+cb^2)$

\\\\\\\\\\\\\\\\\\\\\\\\\\\\\\\\\\\\\\\\\\\\\\\\\\\\\\\\ $\leq(4-abc)(abc+ab^2+cb^2)$

\\\\\\\\\\\\\\\\\\\\\\\\\\\\\\\\\\\\\\\\\\\\\\\\\\\\\\\\ $\leq \left [ \frac{b^2(a+c)+4}{2} \right ]^2$

\\\\\\\\\\\\\\\\\\\\\\\\\\\\\\\\\\\\\\\\\\\\\\\\\\\\\\\\ $\leq 16$

( vì $b^2(a+c)=4.\frac{b}{2}.\frac{b}{2}.(a+c)\leq 4\left (\frac{a+b+c}{3} \right )^3=4$ )



#372442 $$2^{3^x} + 1 = 19.3^y$$

Đã gửi bởi kerry0111 on 25-11-2012 - 13:45 trong Số học

dễ thấy pt có no nguyên thì đó phải là no nguyên dương

và theo zsigmondy thì $2^k-(-1)^k\not\equiv 0(mod19)$ với mọi $k=\overline{1;3^x-1}$

mà $2^9-(-1)^9\equiv 0(mod19)$

nên $3^x-1\leq 8\Rightarrow x\leq 2$

từ đó dễ dàng suy ra no của pt trên là $(x, y)$ bằng $(2;3)$



#371775 Tìm min P=OA'.OB'.OC'

Đã gửi bởi kerry0111 on 23-11-2012 - 16:52 trong Hình học

$OA'.BC=OB.CA'+OC.BA'$

$\Rightarrow OA'.BC= R.\frac{BC.sinCOA+BC.sinBOA}{sinBOC}$

$\Rightarrow OA'= R.\frac{sinCOA+sinBOA}{sinBOC}$

từ đó dễ dàng suy ra $P_{min}=8R^3$

dấu "=" xảy ra $\Leftrightarrow \Delta ABC$ đều